0 Daumen
762 Aufrufe

ich muss zeigen, dass der Grenzwert endlich ist. Leider komme ich hier aber nicht sehr weit. Ich hoffe, es kann mir hier jemand helfen.

\( \lim\limits_{k\to\infty} \frac{\binom{n}{k}}{k^{-1-n}}\)

Meine Überlegung:

\( \lim\limits_{k\to\infty} \frac{\binom{n}{k}}{k^{-1-n}}\)  = \( \lim\limits_{k\to\infty} \frac{\frac{n \cdot \dots \cdot (n-k+1) }{k!}}{k^{-1-n}}\) = \( \lim\limits_{k\to\infty} \frac{n \cdot \dots \cdot (n-k+1) }{k! k^{-1-n}}\)


Ab hier komm ich leider nicht weiter.

Vielen Dank schon mal :)

Avatar von

1 Antwort

0 Daumen

Nur mal zur Klarstellung: k geht gegen unendlich, während n fest und endlich ist?

Irgendwann ist also k>n?

Avatar von 54 k 🚀

Ich sollte zeigen, dass es in O(k^-1-n) ist, also bin ich beim Umschreiben davon ausgegangen. n ist eine reelle Zahl größer gleich 0

Damit gehst du zielsicher an meinen Bedenken vorbei (und erzeugst neue Bedenken).

n ist eine reelle Zahl größer gleich 0

n ist also nicht mal zwingend ganzzahlig?


also bin ich beim Umschreiben davon ausgegangen


Das klingt nach: "Ich habe meine Eigeninterpretation der Aufgabe gepostet"

Gibt es auch eine Originalversion der Aufgabe?

Sei \( n \in \mathbb{R} \) mit \( n \geq 0 \) . Zeigen Sie \( \binom{n}{k} \in O(k^{-1-n})\) .

Tut mir leid, ich muss passen.

Was ist eigentlich mit

O(k^-1-n)

gemeint?

Also mal eine Definition von O:

\( f \in O(g) \Leftrightarrow lim _{n \rightarrow \infty} sup \frac{f(n)}{g(n)} < \infty\) .

Deshalb auch meine Umschreibung. Entschuldige für die Verwirrung.

Tut mir leid, auch jetzt fehlt mir das notwendige Wissen. Ich habe auch keine Ahnung, wieso du im Rahmen der jetzt gelieferten Definition offensichtlich für g(x) den Term \(k^{-1-n}\) eingesetzt hast.

Gibt es auch eine verborgene Definition für g(x)?

Ein anderes Problem?

Stell deine Frage

Willkommen bei der Mathelounge! Stell deine Frage einfach und kostenlos

x
Made by a lovely community